Difference between revisions of "2021 Fall AMC 10A Problems/Problem 12"

(Corrected the sign error, and reformatted. Made Sol 1 more concise.)
(Tag: New redirect)
 
(2 intermediate revisions by the same user not shown)
Line 1: Line 1:
==Problem==
+
#REDIRECT [[2021_Fall_AMC_12A_Problems/Problem_10]]
The base-nine representation of the number <math>N</math> is <math>27{,}006{,}000{,}052_{\text{nine}}.</math> What is the remainder when <math>N</math> is divided by <math>5?</math>
 
 
 
<math>\textbf{(A) } 0\qquad\textbf{(B) } 1\qquad\textbf{(C) } 2\qquad\textbf{(D) } 3\qquad\textbf{(E) }4</math>
 
 
 
==Solution 1==
 
Recall that <math>9\equiv-1\pmod{5}.</math> We have
 
<cmath>\begin{align*}
 
27{,}006{,}000{,}052_9 &= 2\cdot9^{10} + 7\cdot9^9 + 6\cdot9^6 + 5\cdot9 + 2 \\
 
&\equiv 2\cdot(-1)^{10} + 7\cdot(-1)^9 + 6\cdot(-1)^6 + 5\cdot(-1) + 2 &&\pmod{5} \\
 
&= 2-7+6-5+2 \\
 
&= -2 \\
 
&\equiv \boxed{\textbf{(D) } 3} &&\pmod{5}.
 
\end{align*}</cmath>
 
-Aidensharp ~MRENTHUSIASM
 
 
 
==Solution 2==
 
We convert this into base <math>10,</math> so
 
<cmath>2 \cdot 9^{10}+7 \cdot 9^9+6 \cdot 9^6+5 \cdot 9+2</cmath>
 
Notice that <math>9 \equiv -1 \mod 5,</math>
 
<cmath>2 \cdot (-1)^10+7 \cdot (-1)^9+6 \cdot (-1)^6+5 \cdot (-1)+2=2-7+6-5+2</cmath>
 
Simplifying, <math>-2 \mod 5 \implies 3 \mod 5.</math> So, the answer is <math>\boxed{3}.</math>
 
 
 
-kante314
 
 
 
==See Also==
 
{{AMC10 box|year=2021 Fall|ab=A|num-b=11|num-a=13}}
 
{{MAA Notice}}
 

Latest revision as of 20:48, 23 November 2021